You are on page 1of 3

MATH 104: INTRODUCTORY ANALYSIS SPRING 2008/09 PROBLEM SET 3 SOLUTIONS

1. (a) Prove Bernoullis inequality: (1 + x)n 1 + nx for every real number x 1 and every n N. Solution. This is clearly true for n = 1. Suppose it is true for n = k. Then for n = k +1, we have (1 + x)k+1 = (1 + x)k (1 + x) (1 + kx)(1 + x) = 1 + kx + x + kx2 1 + (k + 1)x for any x 1. Hence this is true for all n N by the principle of mathematical induction. (b) Dene the sequence (an )nN and (bn )nN by an = 2 1 2n and bn = 2n2 + 1 . n2 + 3n

Prove from the denition of limits that


n

lim an = 2 = lim bn .
n

In other words, given > 0, you should produce a corresponding N N that satises the denition. Solution. By Bernoullis inequality, |an 2| = 1 1 1 = . 2n (1 + 1)n 1+n

Hence |an 1| < 1/n for every n N. By the Archimedean property, given any > 0 there is an N N such that 1/N < . Now if n > N , we have |an 2| < and so limn an = 2. Let > 0 be given. Then 2n2 + 1 2n2 + 1 2(n2 + 3n) 2 = 2 + 3n n n2 + 3n 1 6n = 2 n + 3n 6n 1 6n 6 = 2 < 2 = . n + 3n n n Now by the Archimedean property, we may choose N N such that 1/N < /6 (or alternatively, let N = 6/ N). Then for n > N we have 2n2 + 1 6 6 2 < < < . n2 + 3n n N 2. Let (an )nN and (bn )nN be sequences of real numbers. (a) Suppose limn an = 0 and (bn )nN is bounded (but not necessarily convergent). Prove that limn an bn = 0.
Date: March 4, 2009 (Version 1.0).
1

1 1 < < n N

Solution. Since (bn )nN is bounded, there exists M > 0 such that |bn | M for all n N. Let > 0 be given. Since limn an = 0, there exists N N such that |an 0| < whenever n > N . Hence |an bn 0| |an ||bn | < whenever n > N . In other words,
n

M = M

lim an bn = 0.

(b) Suppose (an )nN is convergent and (bn )nN is divergent. Prove that (an +bn )nN is divergent. Solution. Suppose (an + bn )nN is convergent. Then by Theorem 1.2 in the lectures, (bn )nN = (an + bn )nN (an )nN is also convergent. A contradiction to the assumption that (bn )nN is divergent. 3. Let (an )nN and (bn )nN be sequences of real numbers. Are the following statements true or false? You need to prove the statement or give a counterexample. (a) If (an )nN is convergent and (bn )nN is divergent, then (an bn )nN is divergent. Solution. False. A counter example is given by an = 1/n and bn = (1)n . Note that limn an bn = limn (1)n /n = 0. (b) If (an )nN and (bn )nN are both divergent, then (an + bn )nN is divergent. Solution. False. Take any divergent (an )nN and let (bn )nN = (an )nN and so limn an + bn = limn 0 = 1. (c) If (an )nN and (bn )nN are both divergent, then (an bn )nN is divergent. Solution. False. A counter example is given by an = (1)n = bn . Note that limn an bn = limn 1 = 1. 4. Let (an )nN be a sequence of real numbers. Dene the sequence (sn )nN by sn := a1 + a2 + + an n

for every n N. (a) Prove that if limn an = a, then limn sn = a. Solution. Let > 0 be given. Since limn an = a, there exists N1 N such that |an a| < /2 whenever n > N1 . Now by the Archimedean property, there exists N2 N such that |a1 a| + |a2 a| + + |aN1 a| < . N2 2 Hence |a1 a| + |a2 a| + + |aN1 a| < n 2
2

whenever n > N2 . Now for n > max{N1 , N2 }, a1 + a2 + + an a n (a1 a) + (a2 a) + + (an a) = n |a1 a| + |a2 a| + + |an a| n |a1 a| + + |aN1 a| |aN1 +1 a| + + |an a| + = n n n N1 < + < . 2 n 2 Hence limn sn = a. (b) Give an example to show that the converse is not always true. Solution. Let an = (1)n . Then (an )nN is divergent. However observe that 1 if n odd, sn = n 0 if n even, |sn a| = and so (sn )nN is convergent. In fact limn sn = 0.

You might also like